How to determine parameters $a, b,$ and $d$ so that a rational function models a given graph?

The name of the pictureThe name of the pictureThe name of the pictureClash Royale CLAN TAG#URR8PPP











up vote
5
down vote

favorite
2












I'm trying to solve a function problem. It states:



Determine the values of $a$, $b$, and $d$ so that the rational function $$f(x) = frac(x+a)(x-1)(x-b)(x-c)(x+d)(x-3)$$



correctly models this graph:



enter image description here



I've been looking at this for a while and I just can't figure out how I'm supposed to approach it. I think it has something to do with limits and I know they're all going to be integer numbers. Any ideas beyond randomly plugging numbers into Desmos until I get the right graph?







share|cite|improve this question

















  • 6




    You should immediately see that, according to the graph, the function is not defined at $x=-1$ and $x=2$ and at these points the graph goes to (plus or minus) infinity. Therefore, the resulting function should have those as the zeros of the denominator. Furthermore, the function is also not defined at $x=3$, as we can also see from the equation form. But this should "cancel out", as the function does not go to +/- infinity when approaching this point. These points should already get you pretty far ...
    – Matti P.
    Aug 2 at 7:41






  • 2




    ... and the zeros of the function should correspond to zeros of the numerator. There are two of them on the graph, but the numerator has three zeros, so something should cancel out. The remaining consideration is the limits as x goes to $pminfty$.
    – NickD
    Aug 2 at 12:37











  • Thank you to everyone who provided help on this question. It not only helped me solve it but provided much-needed insight for the future!
    – CaptainAmerica16
    Aug 3 at 17:05














up vote
5
down vote

favorite
2












I'm trying to solve a function problem. It states:



Determine the values of $a$, $b$, and $d$ so that the rational function $$f(x) = frac(x+a)(x-1)(x-b)(x-c)(x+d)(x-3)$$



correctly models this graph:



enter image description here



I've been looking at this for a while and I just can't figure out how I'm supposed to approach it. I think it has something to do with limits and I know they're all going to be integer numbers. Any ideas beyond randomly plugging numbers into Desmos until I get the right graph?







share|cite|improve this question

















  • 6




    You should immediately see that, according to the graph, the function is not defined at $x=-1$ and $x=2$ and at these points the graph goes to (plus or minus) infinity. Therefore, the resulting function should have those as the zeros of the denominator. Furthermore, the function is also not defined at $x=3$, as we can also see from the equation form. But this should "cancel out", as the function does not go to +/- infinity when approaching this point. These points should already get you pretty far ...
    – Matti P.
    Aug 2 at 7:41






  • 2




    ... and the zeros of the function should correspond to zeros of the numerator. There are two of them on the graph, but the numerator has three zeros, so something should cancel out. The remaining consideration is the limits as x goes to $pminfty$.
    – NickD
    Aug 2 at 12:37











  • Thank you to everyone who provided help on this question. It not only helped me solve it but provided much-needed insight for the future!
    – CaptainAmerica16
    Aug 3 at 17:05












up vote
5
down vote

favorite
2









up vote
5
down vote

favorite
2






2





I'm trying to solve a function problem. It states:



Determine the values of $a$, $b$, and $d$ so that the rational function $$f(x) = frac(x+a)(x-1)(x-b)(x-c)(x+d)(x-3)$$



correctly models this graph:



enter image description here



I've been looking at this for a while and I just can't figure out how I'm supposed to approach it. I think it has something to do with limits and I know they're all going to be integer numbers. Any ideas beyond randomly plugging numbers into Desmos until I get the right graph?







share|cite|improve this question













I'm trying to solve a function problem. It states:



Determine the values of $a$, $b$, and $d$ so that the rational function $$f(x) = frac(x+a)(x-1)(x-b)(x-c)(x+d)(x-3)$$



correctly models this graph:



enter image description here



I've been looking at this for a while and I just can't figure out how I'm supposed to approach it. I think it has something to do with limits and I know they're all going to be integer numbers. Any ideas beyond randomly plugging numbers into Desmos until I get the right graph?









share|cite|improve this question












share|cite|improve this question




share|cite|improve this question








edited Aug 3 at 9:44









TheSimpliFire

9,40951751




9,40951751









asked Aug 2 at 7:36









CaptainAmerica16

154110




154110







  • 6




    You should immediately see that, according to the graph, the function is not defined at $x=-1$ and $x=2$ and at these points the graph goes to (plus or minus) infinity. Therefore, the resulting function should have those as the zeros of the denominator. Furthermore, the function is also not defined at $x=3$, as we can also see from the equation form. But this should "cancel out", as the function does not go to +/- infinity when approaching this point. These points should already get you pretty far ...
    – Matti P.
    Aug 2 at 7:41






  • 2




    ... and the zeros of the function should correspond to zeros of the numerator. There are two of them on the graph, but the numerator has three zeros, so something should cancel out. The remaining consideration is the limits as x goes to $pminfty$.
    – NickD
    Aug 2 at 12:37











  • Thank you to everyone who provided help on this question. It not only helped me solve it but provided much-needed insight for the future!
    – CaptainAmerica16
    Aug 3 at 17:05












  • 6




    You should immediately see that, according to the graph, the function is not defined at $x=-1$ and $x=2$ and at these points the graph goes to (plus or minus) infinity. Therefore, the resulting function should have those as the zeros of the denominator. Furthermore, the function is also not defined at $x=3$, as we can also see from the equation form. But this should "cancel out", as the function does not go to +/- infinity when approaching this point. These points should already get you pretty far ...
    – Matti P.
    Aug 2 at 7:41






  • 2




    ... and the zeros of the function should correspond to zeros of the numerator. There are two of them on the graph, but the numerator has three zeros, so something should cancel out. The remaining consideration is the limits as x goes to $pminfty$.
    – NickD
    Aug 2 at 12:37











  • Thank you to everyone who provided help on this question. It not only helped me solve it but provided much-needed insight for the future!
    – CaptainAmerica16
    Aug 3 at 17:05







6




6




You should immediately see that, according to the graph, the function is not defined at $x=-1$ and $x=2$ and at these points the graph goes to (plus or minus) infinity. Therefore, the resulting function should have those as the zeros of the denominator. Furthermore, the function is also not defined at $x=3$, as we can also see from the equation form. But this should "cancel out", as the function does not go to +/- infinity when approaching this point. These points should already get you pretty far ...
– Matti P.
Aug 2 at 7:41




You should immediately see that, according to the graph, the function is not defined at $x=-1$ and $x=2$ and at these points the graph goes to (plus or minus) infinity. Therefore, the resulting function should have those as the zeros of the denominator. Furthermore, the function is also not defined at $x=3$, as we can also see from the equation form. But this should "cancel out", as the function does not go to +/- infinity when approaching this point. These points should already get you pretty far ...
– Matti P.
Aug 2 at 7:41




2




2




... and the zeros of the function should correspond to zeros of the numerator. There are two of them on the graph, but the numerator has three zeros, so something should cancel out. The remaining consideration is the limits as x goes to $pminfty$.
– NickD
Aug 2 at 12:37





... and the zeros of the function should correspond to zeros of the numerator. There are two of them on the graph, but the numerator has three zeros, so something should cancel out. The remaining consideration is the limits as x goes to $pminfty$.
– NickD
Aug 2 at 12:37













Thank you to everyone who provided help on this question. It not only helped me solve it but provided much-needed insight for the future!
– CaptainAmerica16
Aug 3 at 17:05




Thank you to everyone who provided help on this question. It not only helped me solve it but provided much-needed insight for the future!
– CaptainAmerica16
Aug 3 at 17:05










3 Answers
3






active

oldest

votes

















up vote
13
down vote



accepted










from the expression, we see that whenever the numerator is zero, the graph intersects $x$ axis, and the numerator becomes zero when either of the factors is zero hence possible roots:
$x=-a, x=1$ and $x=b $. Also when the denominator is zero, the graph should tend to infinity if the numerator is non-zero at the same time. ie:
$x= c, x=-d$ and $x=3$.



From graph, roots are $x=-2,x=1$. Hence hints for $a$ and $b$.



Also as the discontinuity is at $x=3$, the function is not defined there hence the denominator is zero but the numerator is also but because nothing can be divided by zero, it is not defined at that single point. So you have to think of ways in which we can choose $c$ and $d$ such that at one of the function is not defined and at the other it simply reaches infinity.



You can take it from here, I guess.






share|cite|improve this answer



















  • 2




    Welcome to the Mathematics Stack Exchange (Math.SE)! I must say, 'tis a beautiful answer :D
    – user477343
    Aug 3 at 9:47


















up vote
7
down vote













HINT:



  • You can see that at $x=-1,2$, $f$ goes to infinity so $c$ and $d$ are ?


  • Then use the points $(-2,0)$ and $(1,0)$ to solve for $a$ and $b$.






share|cite|improve this answer




























    up vote
    3
    down vote













    For this sort of problem, you should start by looking at "special points": intercepts, asymptotes, discontinuities, etc. You can then move on to looking for those for the derivative, second derivative, etc.



    There are three special points highlighted for you: $x$-intercepts at $-2$ and $1$, and $y$-intercept at $1$, and a discontinuity at $(3,2.5)$. The $x$-intercepts tell you where the function should be zero: plugging in both $x = -2$ and $x = 1$ should give zero. If you plug $x = 1$ into $x - 1$, that gives you zero, so that's already taken care of. So now you need one of the parentheses to give you zero at $x = -2$. If $x = -2$, then adding $2$ to $x$ gives zero, so you want an $x+2$ in one of the parentheses, so $a = 2$.



    For the $y$-intercept, you can plug in $x = 0$, and see that the equation gives you $$frac(0+2)(0-1)(0-b)(0-c)(0+d)(0-3).$$ This has three unknowns, so it's not very useful.



    We can next look at the vertical asymptotes. There are two of them, at $x = -1$ and $x = 2$. A vertical asymptote corresponds to the denominator being zero, so we should have $x + 2$ and $x - 1$ on the bottom, giving $c = 1$ and $d = 2$. However, there's also an $x - 3$ on the bottom, but no vertical asymptote at $x = 3$. So we need to cancel that out with an $x - 3$ on the top, giving $b = 3$.






    share|cite|improve this answer























      Your Answer




      StackExchange.ifUsing("editor", function ()
      return StackExchange.using("mathjaxEditing", function ()
      StackExchange.MarkdownEditor.creationCallbacks.add(function (editor, postfix)
      StackExchange.mathjaxEditing.prepareWmdForMathJax(editor, postfix, [["$", "$"], ["\\(","\\)"]]);
      );
      );
      , "mathjax-editing");

      StackExchange.ready(function()
      var channelOptions =
      tags: "".split(" "),
      id: "69"
      ;
      initTagRenderer("".split(" "), "".split(" "), channelOptions);

      StackExchange.using("externalEditor", function()
      // Have to fire editor after snippets, if snippets enabled
      if (StackExchange.settings.snippets.snippetsEnabled)
      StackExchange.using("snippets", function()
      createEditor();
      );

      else
      createEditor();

      );

      function createEditor()
      StackExchange.prepareEditor(
      heartbeatType: 'answer',
      convertImagesToLinks: true,
      noModals: false,
      showLowRepImageUploadWarning: true,
      reputationToPostImages: 10,
      bindNavPrevention: true,
      postfix: "",
      noCode: true, onDemand: true,
      discardSelector: ".discard-answer"
      ,immediatelyShowMarkdownHelp:true
      );



      );








       

      draft saved


      draft discarded


















      StackExchange.ready(
      function ()
      StackExchange.openid.initPostLogin('.new-post-login', 'https%3a%2f%2fmath.stackexchange.com%2fquestions%2f2869792%2fhow-to-determine-parameters-a-b-and-d-so-that-a-rational-function-models-a%23new-answer', 'question_page');

      );

      Post as a guest






























      3 Answers
      3






      active

      oldest

      votes








      3 Answers
      3






      active

      oldest

      votes









      active

      oldest

      votes






      active

      oldest

      votes








      up vote
      13
      down vote



      accepted










      from the expression, we see that whenever the numerator is zero, the graph intersects $x$ axis, and the numerator becomes zero when either of the factors is zero hence possible roots:
      $x=-a, x=1$ and $x=b $. Also when the denominator is zero, the graph should tend to infinity if the numerator is non-zero at the same time. ie:
      $x= c, x=-d$ and $x=3$.



      From graph, roots are $x=-2,x=1$. Hence hints for $a$ and $b$.



      Also as the discontinuity is at $x=3$, the function is not defined there hence the denominator is zero but the numerator is also but because nothing can be divided by zero, it is not defined at that single point. So you have to think of ways in which we can choose $c$ and $d$ such that at one of the function is not defined and at the other it simply reaches infinity.



      You can take it from here, I guess.






      share|cite|improve this answer



















      • 2




        Welcome to the Mathematics Stack Exchange (Math.SE)! I must say, 'tis a beautiful answer :D
        – user477343
        Aug 3 at 9:47















      up vote
      13
      down vote



      accepted










      from the expression, we see that whenever the numerator is zero, the graph intersects $x$ axis, and the numerator becomes zero when either of the factors is zero hence possible roots:
      $x=-a, x=1$ and $x=b $. Also when the denominator is zero, the graph should tend to infinity if the numerator is non-zero at the same time. ie:
      $x= c, x=-d$ and $x=3$.



      From graph, roots are $x=-2,x=1$. Hence hints for $a$ and $b$.



      Also as the discontinuity is at $x=3$, the function is not defined there hence the denominator is zero but the numerator is also but because nothing can be divided by zero, it is not defined at that single point. So you have to think of ways in which we can choose $c$ and $d$ such that at one of the function is not defined and at the other it simply reaches infinity.



      You can take it from here, I guess.






      share|cite|improve this answer



















      • 2




        Welcome to the Mathematics Stack Exchange (Math.SE)! I must say, 'tis a beautiful answer :D
        – user477343
        Aug 3 at 9:47













      up vote
      13
      down vote



      accepted







      up vote
      13
      down vote



      accepted






      from the expression, we see that whenever the numerator is zero, the graph intersects $x$ axis, and the numerator becomes zero when either of the factors is zero hence possible roots:
      $x=-a, x=1$ and $x=b $. Also when the denominator is zero, the graph should tend to infinity if the numerator is non-zero at the same time. ie:
      $x= c, x=-d$ and $x=3$.



      From graph, roots are $x=-2,x=1$. Hence hints for $a$ and $b$.



      Also as the discontinuity is at $x=3$, the function is not defined there hence the denominator is zero but the numerator is also but because nothing can be divided by zero, it is not defined at that single point. So you have to think of ways in which we can choose $c$ and $d$ such that at one of the function is not defined and at the other it simply reaches infinity.



      You can take it from here, I guess.






      share|cite|improve this answer















      from the expression, we see that whenever the numerator is zero, the graph intersects $x$ axis, and the numerator becomes zero when either of the factors is zero hence possible roots:
      $x=-a, x=1$ and $x=b $. Also when the denominator is zero, the graph should tend to infinity if the numerator is non-zero at the same time. ie:
      $x= c, x=-d$ and $x=3$.



      From graph, roots are $x=-2,x=1$. Hence hints for $a$ and $b$.



      Also as the discontinuity is at $x=3$, the function is not defined there hence the denominator is zero but the numerator is also but because nothing can be divided by zero, it is not defined at that single point. So you have to think of ways in which we can choose $c$ and $d$ such that at one of the function is not defined and at the other it simply reaches infinity.



      You can take it from here, I guess.







      share|cite|improve this answer















      share|cite|improve this answer



      share|cite|improve this answer








      edited Aug 3 at 9:45









      TheSimpliFire

      9,40951751




      9,40951751











      answered Aug 2 at 7:51









      Zulu Raman

      1484




      1484







      • 2




        Welcome to the Mathematics Stack Exchange (Math.SE)! I must say, 'tis a beautiful answer :D
        – user477343
        Aug 3 at 9:47













      • 2




        Welcome to the Mathematics Stack Exchange (Math.SE)! I must say, 'tis a beautiful answer :D
        – user477343
        Aug 3 at 9:47








      2




      2




      Welcome to the Mathematics Stack Exchange (Math.SE)! I must say, 'tis a beautiful answer :D
      – user477343
      Aug 3 at 9:47





      Welcome to the Mathematics Stack Exchange (Math.SE)! I must say, 'tis a beautiful answer :D
      – user477343
      Aug 3 at 9:47











      up vote
      7
      down vote













      HINT:



      • You can see that at $x=-1,2$, $f$ goes to infinity so $c$ and $d$ are ?


      • Then use the points $(-2,0)$ and $(1,0)$ to solve for $a$ and $b$.






      share|cite|improve this answer

























        up vote
        7
        down vote













        HINT:



        • You can see that at $x=-1,2$, $f$ goes to infinity so $c$ and $d$ are ?


        • Then use the points $(-2,0)$ and $(1,0)$ to solve for $a$ and $b$.






        share|cite|improve this answer























          up vote
          7
          down vote










          up vote
          7
          down vote









          HINT:



          • You can see that at $x=-1,2$, $f$ goes to infinity so $c$ and $d$ are ?


          • Then use the points $(-2,0)$ and $(1,0)$ to solve for $a$ and $b$.






          share|cite|improve this answer













          HINT:



          • You can see that at $x=-1,2$, $f$ goes to infinity so $c$ and $d$ are ?


          • Then use the points $(-2,0)$ and $(1,0)$ to solve for $a$ and $b$.







          share|cite|improve this answer













          share|cite|improve this answer



          share|cite|improve this answer











          answered Aug 2 at 7:42









          TheSimpliFire

          9,40951751




          9,40951751




















              up vote
              3
              down vote













              For this sort of problem, you should start by looking at "special points": intercepts, asymptotes, discontinuities, etc. You can then move on to looking for those for the derivative, second derivative, etc.



              There are three special points highlighted for you: $x$-intercepts at $-2$ and $1$, and $y$-intercept at $1$, and a discontinuity at $(3,2.5)$. The $x$-intercepts tell you where the function should be zero: plugging in both $x = -2$ and $x = 1$ should give zero. If you plug $x = 1$ into $x - 1$, that gives you zero, so that's already taken care of. So now you need one of the parentheses to give you zero at $x = -2$. If $x = -2$, then adding $2$ to $x$ gives zero, so you want an $x+2$ in one of the parentheses, so $a = 2$.



              For the $y$-intercept, you can plug in $x = 0$, and see that the equation gives you $$frac(0+2)(0-1)(0-b)(0-c)(0+d)(0-3).$$ This has three unknowns, so it's not very useful.



              We can next look at the vertical asymptotes. There are two of them, at $x = -1$ and $x = 2$. A vertical asymptote corresponds to the denominator being zero, so we should have $x + 2$ and $x - 1$ on the bottom, giving $c = 1$ and $d = 2$. However, there's also an $x - 3$ on the bottom, but no vertical asymptote at $x = 3$. So we need to cancel that out with an $x - 3$ on the top, giving $b = 3$.






              share|cite|improve this answer



























                up vote
                3
                down vote













                For this sort of problem, you should start by looking at "special points": intercepts, asymptotes, discontinuities, etc. You can then move on to looking for those for the derivative, second derivative, etc.



                There are three special points highlighted for you: $x$-intercepts at $-2$ and $1$, and $y$-intercept at $1$, and a discontinuity at $(3,2.5)$. The $x$-intercepts tell you where the function should be zero: plugging in both $x = -2$ and $x = 1$ should give zero. If you plug $x = 1$ into $x - 1$, that gives you zero, so that's already taken care of. So now you need one of the parentheses to give you zero at $x = -2$. If $x = -2$, then adding $2$ to $x$ gives zero, so you want an $x+2$ in one of the parentheses, so $a = 2$.



                For the $y$-intercept, you can plug in $x = 0$, and see that the equation gives you $$frac(0+2)(0-1)(0-b)(0-c)(0+d)(0-3).$$ This has three unknowns, so it's not very useful.



                We can next look at the vertical asymptotes. There are two of them, at $x = -1$ and $x = 2$. A vertical asymptote corresponds to the denominator being zero, so we should have $x + 2$ and $x - 1$ on the bottom, giving $c = 1$ and $d = 2$. However, there's also an $x - 3$ on the bottom, but no vertical asymptote at $x = 3$. So we need to cancel that out with an $x - 3$ on the top, giving $b = 3$.






                share|cite|improve this answer

























                  up vote
                  3
                  down vote










                  up vote
                  3
                  down vote









                  For this sort of problem, you should start by looking at "special points": intercepts, asymptotes, discontinuities, etc. You can then move on to looking for those for the derivative, second derivative, etc.



                  There are three special points highlighted for you: $x$-intercepts at $-2$ and $1$, and $y$-intercept at $1$, and a discontinuity at $(3,2.5)$. The $x$-intercepts tell you where the function should be zero: plugging in both $x = -2$ and $x = 1$ should give zero. If you plug $x = 1$ into $x - 1$, that gives you zero, so that's already taken care of. So now you need one of the parentheses to give you zero at $x = -2$. If $x = -2$, then adding $2$ to $x$ gives zero, so you want an $x+2$ in one of the parentheses, so $a = 2$.



                  For the $y$-intercept, you can plug in $x = 0$, and see that the equation gives you $$frac(0+2)(0-1)(0-b)(0-c)(0+d)(0-3).$$ This has three unknowns, so it's not very useful.



                  We can next look at the vertical asymptotes. There are two of them, at $x = -1$ and $x = 2$. A vertical asymptote corresponds to the denominator being zero, so we should have $x + 2$ and $x - 1$ on the bottom, giving $c = 1$ and $d = 2$. However, there's also an $x - 3$ on the bottom, but no vertical asymptote at $x = 3$. So we need to cancel that out with an $x - 3$ on the top, giving $b = 3$.






                  share|cite|improve this answer















                  For this sort of problem, you should start by looking at "special points": intercepts, asymptotes, discontinuities, etc. You can then move on to looking for those for the derivative, second derivative, etc.



                  There are three special points highlighted for you: $x$-intercepts at $-2$ and $1$, and $y$-intercept at $1$, and a discontinuity at $(3,2.5)$. The $x$-intercepts tell you where the function should be zero: plugging in both $x = -2$ and $x = 1$ should give zero. If you plug $x = 1$ into $x - 1$, that gives you zero, so that's already taken care of. So now you need one of the parentheses to give you zero at $x = -2$. If $x = -2$, then adding $2$ to $x$ gives zero, so you want an $x+2$ in one of the parentheses, so $a = 2$.



                  For the $y$-intercept, you can plug in $x = 0$, and see that the equation gives you $$frac(0+2)(0-1)(0-b)(0-c)(0+d)(0-3).$$ This has three unknowns, so it's not very useful.



                  We can next look at the vertical asymptotes. There are two of them, at $x = -1$ and $x = 2$. A vertical asymptote corresponds to the denominator being zero, so we should have $x + 2$ and $x - 1$ on the bottom, giving $c = 1$ and $d = 2$. However, there's also an $x - 3$ on the bottom, but no vertical asymptote at $x = 3$. So we need to cancel that out with an $x - 3$ on the top, giving $b = 3$.







                  share|cite|improve this answer















                  share|cite|improve this answer



                  share|cite|improve this answer








                  edited Aug 3 at 9:43









                  TheSimpliFire

                  9,40951751




                  9,40951751











                  answered Aug 2 at 19:08









                  Acccumulation

                  4,015314




                  4,015314






















                       

                      draft saved


                      draft discarded


























                       


                      draft saved


                      draft discarded














                      StackExchange.ready(
                      function ()
                      StackExchange.openid.initPostLogin('.new-post-login', 'https%3a%2f%2fmath.stackexchange.com%2fquestions%2f2869792%2fhow-to-determine-parameters-a-b-and-d-so-that-a-rational-function-models-a%23new-answer', 'question_page');

                      );

                      Post as a guest













































































                      Comments

                      Popular posts from this blog

                      Color the edges and diagonals of a regular polygon

                      Relationship between determinant of matrix and determinant of adjoint?

                      What is the equation of a 3D cone with generalised tilt?